MRCP revision for free - Home

Question category: Cardiology

A 65 year old male is found incidentally to have an ejection systolic murmur. An Echo confirms aortic stenosis with a gradient of 80mmHg and left ventricular dysfunction. How should this patient be managed?


Please log in to record your progress.


Sign up for free to track your progress